Determine all composite integers $n>1$ that satisfy the following property: if $d_1$, $d_2$, $\ldots$, $d_k$ are all the positive divisors of $n$ with $1 = d_1 < d_2 < \cdots < d_k = n$, then $d_i$ divides $d_{i+1} + d_{i+2}$ for every $1 \leq i \leq k - 2$.
Problem
Source: IMO 2023 P1
Tags: IMO, IMO 2023, number theory, Divisors
08.07.2023 08:00
Assume $p<q$ be the smallest two prime divisors of $n$. Then $d_{k-1}=\frac{n}{p}$. Assume $d_m=\frac{n}{q}$ for some $m$, and $d_{m+1}=\frac{n}{p^{c+1}}$ and $d_{m+2}=\frac{n}{p^c}$ for some nonnegative integer $c$. Then, since $d_m\mid d_{m+1}+d_{m+2}$, $p^{c+1}\mid q+pq$ and $p|q$ which is a contradiction. Therefore $n$ does not have two distinct prime factors; $n$=$p^t$ for some prime $p$ and a positive integer $t\neq1$. It's easy to show that this suffices.
08.07.2023 08:02
08.07.2023 08:14
From $d_{k-2} \mid d_{k-1} + d_{k}$ one can get that $d_{k-2} \mid d_{k-1}$ since $d_{k-2} \mid n = d_k$ Then $d_2 = \frac{n}{d_{k-1}} \mid \frac{n}{d_{k-2}} = d_3$ so $d_2 \mid d_3$. Here, since $d_2 \mid d_3 + d_4$, we get that $d_2 \mid d_4$. Again, we can easily show $d_{k-3} | d_{k-1}$ in the same way, which leads to $d_{k-3} \mid d_{k-2}$ By induction, $d_1 \mid d_2 \mid \cdots \mid d_k$ If $p \mid n$ and $q \mid n$ for prime $p > q$ $\frac{n}{p} \mid \frac{n}{q}$ so $q \mid p$ and this is contradiction Hence, $n = p^{k-1}$ This indeed fits the condition
08.07.2023 08:15
The answer is prime powers, which obviously work. Note that the three largest divisors of $n$ are either $\left \{ \tfrac{n}{q}, \tfrac{n}{p}, n \right \}$ for distinct prime divisors $p$ and $q$ of $n$, or $\left \{ \tfrac{n}{p^2}, \tfrac{n}{p}, n \right \}$ for some prime divisor $p$ of $n$. In the former case we have a contradiction, since \[\frac{\frac{n}{p} + n}{\frac{n}{q}} = \frac{q(p+1)}{p},\]obviously not an integer. So, the three largest divisors of $n$ are $\frac{n}{p^2}, \frac{n}{p}$ and $n$. With similar reasoning, now consider the fourth largest divisor. It is either $\tfrac{n}{q}$ or $\tfrac{n}{p^3}$; the former option fails, since \[ \frac{\frac{n}{p} + \frac{n}{p^2}}{\frac{n}{q}} = \frac{q(p+1)}{p^2}, \]obviously not an integer either. We repeat this reasoning to deduce that $n$ is just $p^{k-1}$.
08.07.2023 08:25
HUH how is this IMO level?
08.07.2023 08:29
The answer is $p^e$ for all primes $p$ and $e>1$, which obviously works. Let $p$ be the smallest prime factor of $n$, we claim that $p$ is the only prime factor of $n$. We use induction downward to prove $d_i = \frac{n}{p^{k-i}}$. The base case is obvious. For the inductive step, if we instead had a different prime $q$, then \[ \frac{n}{q} \mid \frac{n}{p^{k-i}} + \frac{n}{p^{k-i-1}}, \]but this is obviously false. We are done.
08.07.2023 08:34
This NT seems familiar. Maybe it's an old problem?
08.07.2023 08:35
LoloChen wrote: This NT seems familiar. Maybe it's an old problem? The only related problem I can think of is IMO 2002 P4, but it's not even close.
08.07.2023 08:42
LoloChen wrote: This NT seems familiar. Maybe it's an old problem? Maybe JBMO 2002 is similar with this problem but this is even easier than that https://artofproblemsolving.com/community/c6h58610p358021
08.07.2023 08:44
ihatemath123 wrote: The answer is prime powers, which obviously work. Note that the three largest divisors of $n$ are either $\left \{ \frac{n}{q}, \frac{n}{p}, n \right \}$ for distinct prime divisors $p$ and $q$ of $n$, or $\left \{ \frac{n}{p^2}, \frac{n}{p}, n \right \}$ for some prime divisor $p$ of $n$. In the former case we have a contradiction, since \[\frac{\frac{n}{p} + n}{\frac{n}{q}} = \frac{q(p+1)}{p},\]obviously not an integer. So, the three largest divisors of $n$ are $\frac{n}{p^2}, \frac{n}{p}$ and $n$. With similar reasoning, now consider the fourth largest divisor. It is either $\frac{n}{q}$ or $\frac{n}{p^3}$; the former option fails, since \[ \frac{\frac{n}{p} + \frac{n}{p^2}}{\frac{n}{q}} = \frac{q(p+1)}{p^2}, \]obviously not an integer either. We repeat this reasoning to deduce that $n$ is just $p^k$. I think you have a very small mistake isn't it $p^{k-1}$??
08.07.2023 08:50
I'll only show the more technical part below.
08.07.2023 09:05
For any $n$, the 3 largest divisors are $n,\frac{n}{p},\frac{n}{q}$ for primes $p<q$ or $n,\frac{n}{p},\frac{n}{p^2}$. However, first case doesn't work because $\frac{\frac{n}{p}+n}{\frac{n}{q}}=\frac{q}{p}+q$ which is not an integer. Repeat for further cases. All should not work for $q$ as you will get $\frac{q(p+1)}{p^k}$ which are not integers. Thus only $p^{k-1}$ work well there goes 10M N prediction more like 0M now
08.07.2023 09:07
This problem was proposed by me, Santiago Rodriguez from Colombia. I hope that you enjoy it.
08.07.2023 09:08
Far too easy for the IMO. It's just prime powers, which trivially work. Suppose $n$ isn't a prime power. Clearly the list of divisors goes $\left(1,\cdots,\frac{n}{q}, \frac{n}{p^k},\cdots,\frac{n}{p},n\right)$ for some primes $p,q$ and some $k\in\mathbb{N}$. Thus $\frac{n}{q}\mid \frac{n}{p^k}+\frac{n}{p^{k-1}}$ and hence $$\frac{q(p+1)}{p^{k-1}}\in\mathbb{Z}$$Which is impossible unless $k=1$, and for $k=1$ we need $\frac{n+n/p}{n/q}$ to be an integer, which is impossible.
08.07.2023 09:19
yofro wrote: Far too easy for the IMO. It's just prime powers, which trivially work. Suppose $n$ isn't a prime power. Clearly the list of divisors goes $\left(1,\cdots,\frac{n}{q}, \frac{n}{p^k},\cdots,\frac{n}{p},n\right)$ for some primes $p,q$ and some $k\in\mathbb{N}$. Thus $\frac{n}{q}\mid \frac{n}{p^k}+\frac{n}{p^{k-1}}$ and hence $$\frac{q(p+1)}{p^{k-1}}\in\mathbb{Z}$$Which is impossible unless $k=1$, and for $k=1$ we need $\frac{n+n/p}{n/q}$ to be an integer, which is impossible. why $\frac{n}{q} < \frac{n}{p^k}$?
08.07.2023 09:31
Solution : Observe that we have $$d_i | d_{i+1}+d_{i+2} , 1 \leq i \leq k-2$$, then put $i=k-2$ then $$d_{k-2} | d_{k-1}+d_{k}=d_{k-1}+n$$but $$d_{k-2} |n \Longrightarrow d_{k-2} | d_{k-1} .$$Again putting $i=k-3$ we get $$d_{k-3} | d_{k-2}+d_{k-1}$$but on other hand $$d_2=\frac{n}{d_{k-1}} | \frac{n}{d_{k-2}} =d_3 , d_2|d_3+d_4 \Longrightarrow d_2|d_4.$$So $$d_{k-3}|d_{k-1} \Longrightarrow d_{k-3}|d_{k-2} .$$Now using Induction we get $$d_1|d_2|d_3.......d_k$$. Hence $n=p^{k-1}$ is the solution.
08.07.2023 09:32
khan.academy wrote: why $\frac{n}{q} < \frac{n}{p^k}$? I think that the poster meant $k$ is just an natural number not $\tau{n}$ Then choose $a$ such that $d_1 = 1, d_2 = p, d_3 = p^2, \cdots, d_a = p^{a-1}, d_{a+1} = q$ Here, $\frac{n}{q} < \frac{n}{p^a}$
08.07.2023 10:16
I want to post a solution as short as I can. Tell me if I made a mistake. Really too easy to be an IMO problem. Quote: $n=p^a$ triviral. If $ n$ has 2 minimal prime divisors $d_1=p<d_t=q$ and $d_{t-1}=p^m$, $\frac n q \mid \frac n {p^m}+\frac n {p^{m-1}}$. $v_p(LHS)>v_p(RHS)$, which leads to contradiction.
08.07.2023 10:18
$d_{n-2}$ divides $d_{n-1} + d_n$ so $d_{n-2}$ divides $d_{n-1}$ $d_{n-3}$ divides $d_{n-2} + d_{n-1}$ so $d_{n-3}$ divides $d_{n-2}$ repeat to get that $d_a$ divides $d_{a+1}$ where $a$ is random integer So $d_a$ divides $d_{a+1}$, which divides $d_{a+2}$, ...... so $d_a$ divides $d_{a+x}$ where $x$ is integer $> 0$ and anything divides or is divided by anything so $n$ has to be a prime power because if it was $a*b$ where $a$ and $b$ or primes than there will be $a$ and $b$ but $a$ does not divide $b$
17.07.2024 22:03
Last year, I wrote a terribly wrong solution and never bother to fix it A year later, I finally did it. We claim all numbers that work are of the form $p^n.$ These obviously work as $p^k|p^k+1(1+p)$ for all integer $k.$ Now assume FTSOC there is a number with multiple prime factors. Let the two smallest be $p$ and $q.$ (Edit: with p<q) Let $c$ be the largest integer such that $p^c<q.$ Then there will be three numbers in a row that are $$\frac{N}{q}, \frac{N}{p^c}, \frac{N}{p^c-1}.$$By the problem statement, we can write that $$\frac{\frac{N}{p^c}(1+p)}{\frac{N}{q}}$$is an integer, which simplifies to $(1+p)\frac{q}{p^c}$. Since $gcd(p+1,p)=1,$ and $gcd(q,p)=1,$ the fraction cannot be further simplified and since $p$ is a prime the fraction is obviously not an integer. Therefore, there are no numbers with multiple prime factors that work. Q.E.D.
18.07.2024 11:42
Sol:- Clearly $p^k$ where $p$ is a prime number (and $k>1$) works. Now suppose a number is divisible by atleast $2$ primes, we let $p<q$ be its smallest $2$ prime factors. Let $r$ be the greatest number such that $p^r<q$. Then $\frac{n}{q},\frac{n}{p^r},\frac{n}{p^{r-1}}$ are $3$ divisors of $n$ in consecutive order. Thus $\frac{n}{q}|(\frac{n}{p^r}+\frac{n}{p^{r-1}}) \implies p^r|q(p+1)$ which is a clear contradiction . Thus only powers( $\neq 1$) of primes works.
19.07.2024 01:19
19.07.2024 02:16
We claim the solution is all prime powers which clearly work. Assume for the sake of contradiction that are two distinct primes that divide $n$. Call the smallest two prime divisors $p$ and $q$ with $p < q$. However, take the largest power of $p$ that is smaller than $q$. Let this be $p^k$, then take $\left(\frac nq, \frac n{p^k}, \frac{n}{p^{k-1}} \right)$ to get a contradiction.
25.07.2024 18:10
The answer is all prime powers $n$, which clearly work. Otherwise, let $p$ and $q$ be the first and second smallest prime divisors of $n$ and let $v_p(n)=e$. Choosing a specific $i$ yields $$\frac{n}{q}\;|\;\frac{n}{p^{e+1}}+\frac{n}{p^{e}}\Rightarrow p^{e+1}\;|\;(p+1)q$$This can not hold, finishing the problem.
08.08.2024 12:03
Answer : $n=p^x$ when $p$ is a prime and $x>1$ a natural number. It's so easy to see these form of solutions. We should look at for some other solutions. Firstly, assume that $n$ has two or more prime divisors. the smallest prime divisors of $n$ be $p<q$. Investigate the case that $p^x$ when $x>1$ and $p^x<q<p^{x+1}$. We can say, $1=d_1$, $p=d_2$, .... , $p^x = d_{x+1}$, $q =d_{x+2}$. From the given property, $d_x| (d_{x+1}+d_{x+2})$. $=$ $p^{x-1}|(p^x + q)$. Then, $(p^x+q)\equiv0\pmod{p}$ So, $p|q$ is impossible. From this, we can say that $x=1$. Investigate this case : If $x=1$, then $d_1=1 , d_2=p, d_3=q, .... , d_{k-2}=\frac{n}{q} , d_{k-1}=\frac{n}{p}$ and $d_k=n$. $d_{k-2}|(d_{k-1}+d_k) = \frac{n}{q}|(\frac{n}{p} + n)$. So, $p|(p+1)q$. Obviously, it's impossible. $n$ can't have two or more prime divisors. $\square$
18.08.2024 02:11
The answer is $n=\boxed{p^k}$ for primes $p$ and positive integers $k\ge 2$ only. Clearly these work. Note that $d_{k-2}\mid d_{k-1}+d_k$, so $d_{k-2}\mid d_{k-1}$. Since $d_2d_{k-1}=d_3d_{k-2}=n$, we get that $d_2\mid d_3$. Let $p=d_2$, then let $d_3=xp$. If $x\ne p$, $x$ would have been a smaller divisor, so $x=p$ exactly. Then we may induct to show that all terms of $\{d_i\}$ besides $d_1$ are divisible by $p$, meaning that $n$ has no other prime divisors, as desired. $\blacksquare$
18.08.2024 10:18
IMO 2023 p1 We claim that the answer is $n=p^k$, where $p$ is a prime. Let $p_1$ and $p_2$ be the first two primes of $n$. Now let $s$ be the largest number, such that $p_1^s < p_2$ So we have that \[\frac{n}{p_2},\frac{n}{p_1^s},\frac{n}{p_1^{s-1}}\]are consecutive divisors of $n$. Hence we must have that. $$\frac{n}{p_2}|(\frac{n}{p_1^s}+\frac{n}{p_1^{s-1}}) \implies p_1^s|p_2(p_1+1)$$Which is a contradiction, hence $n$ has less than 2 primes.
30.09.2024 04:00
I claim the answer is only prime powers. It is easy to see that these work. To see nothing else works, assume for the sake of contradiction some $n$ with more than $1$ prime power worked. Then take the second smallest prime divisor, let it be $q$, and the largest divisor less than it, $p^{k}$, where $k$ can be $1$. We would then have $\frac nq \mid \frac{n}{p^{k - 1}} + \frac{n}{p^k}$, which is obviously not true by taking $\nu_p$ on both sides, the left side has $\nu_p(\frac nq) = \nu_p(n)$, right hand side has $\nu_p({\frac{n}{p^{k - 1}} + \frac{n}{p^k}}) = \nu_p({\frac{n}{p^k}})$, so the left side cannot divide the right.
05.12.2024 19:27
06.12.2024 03:22
Trivial NT on the IMO. Finally found some time to actually post my solution from the contest. We claim that the answer is all positive integers of the form $p^m$ for some prime $p$ and $m\ge 2$. These clearly all work since for all $1\le i \le m+1$ , $d_i=p^{i-1}$. Thus, \[d_i=p_{i-1}\mid p_i + p_{i+1}=d_{i+1}+d_{i+2}\]quite clearly. Now, we show that no other composite $n$ work. Say $n$ has atleast two distinct primes divisors, of which the smallest two are $p<q$. Note that we have $d_i = p^{i-1}$ for all $1\le i \le r$ for some $r \ge 2$ and $d_{r+1}=q$ as the divisors are arranged in increasing order. Now, if $n$ satisfies the desired condition we must have, \begin{align*} d_{m-r} &\mid d_{m-r+1} + d_{m-r+2}\\ \frac{n}{d_{r+1}} & \mid \frac{n}{d_r} + \frac{n}{d_{r-1}}\\ \frac{n}{q} & \mid \frac{n}{p^{r-1}} + \frac{n}{p^{r-2}}\\ p^{r-1} & \mid q + pq\\ p &\mid q+pq\\ p& \mid q \end{align*}which is a very clear contradiction since $q>p$ are both primes. Thus, there are no such $n$ of this form, which finishes the proof.
02.01.2025 20:13
Clear answer is n =p^c where p is a prime Assume that n can have more than 1 prime factor and assume some prime q also divides n Well you get that p|q contradiction. Too easy for P1 tho
17.01.2025 12:53
My first NT after decades..... IMO 2023 P1 wrote: Determine all composite integers $n>1$ that satisfy the following property: if $d_1$, $d_2$, $\ldots$, $d_k$ are all the positive divisors of $n$ with $1 = d_1 < d_2 < \cdots < d_k = n$, then $d_i$ divides $d_{i+1} + d_{i+2}$ for every $1 \leq i \leq k - 2$. All numbers of the form $p^l$ where $p$ is a prime and $l>1$ work evidently. We now show these are the only ones. Let $m$ denote the number of divisors of $n$. Then we have $d_{i}d_{m+1-i}=n$. Thus, $$d_{i} \mid d_{i+1}+d_{i+2}$$$$\implies \frac{n}{d_{m-i+1}} \mid \frac{n}{d_{m-i}}+\frac{n}{d_{m-i-1}}$$$$\implies d_{m-i}d_{m-i-1} \mid d_{m-i+1}(d_{m-i}+d_{m-i-1})$$$$\implies \boxed{d_{i}d_{i+1} \mid d_{i+2}(d_{i}+d_{i+1})} [\because i \mapsto m-i-1]$$Now $i=1$ gives, $d_{1}d_{2} \mid d_{3}(d_{1}+d_{2}) \implies d_{2} \mid d_{3}$ since $d_{1}=1$. Also, $d_{2}$ is a prime number (say $p$). If $d_{3}$ has any prime factor $q$(say) other than $p$ then $d_{1}<q<d_{3};q \neq d_{2}$ which is a blatant contradiction. Thus $\boxed{d_{3}=p^{2}}$ as it can not be $p^e$ where $e \geq 3$ otherwise $d_{2} < p^{2} < d_{3}$ again, a contradiction. Now by an easy strong induction and similar argument we can conclude that $d_{i+1}=p^{i}$. Thus $n=p^{l}$ where $l>1$ and $p$ is a prime number. $\blacksquare$ ($\mathcal{QED}$)
17.01.2025 13:33
Suppose $n$ has a prime divisors $\geq 2$. Say the least ones are $p$ and $q$ and $p$ be the minimum. Let the multiplicity of $p$ be $m$. At the $(k+1)$-th step, $k \leq m$: $p^{k-1} < q$, so $q$ can write $p^{k-1} \mid p^k + q \quad ( \rightarrow \leftarrow)$. Hence, $n = p^t$, $t \in \mathbb{N}$ and $p$ is prime.
29.01.2025 04:35
Note that since $d_{k-2}\mid d_{k-1}+d_k$ and $d_{k-2}\mid n=d_k$, then $d_{k-2}\mid d_{k-1}$. Recall that $d_{k-2}=\frac{n}{d_3}$ and $d_{k-1}=\frac{n}{d_2}$, so $\frac{\frac{n}{d_2}}{\frac{n}{d_3}}\in\mathbb{N}\Longleftrightarrow\frac{d_3}{d_2}\in\mathbb{N}\Longleftrightarrow d_2\mid d_3$. But we also have $d_2\mid d_3+d_4$, so we must have $d_2\mid d_4\Longleftrightarrow\frac{\frac{n}{d_2}}{\frac{n}{d_4}}\in\mathbb{N}\Longleftrightarrow\frac{d_{k-1}}{d_{k-3}}\in\mathbb{N}\Longleftrightarrow d_{k-3}\mid d_{k-1}$. Again, we know that $d_{k-3}\mid d_{k-2}+d_{k-1}$ so we must have $d_{k-3}\mid d_{k-2}$. Repeating the process, eventually we'll end with $1=d_1\mid d_2\mid d_3\mid\dots\mid d_{k-1}\mid d_k=n$. Notice that for any natural number $n$, the three largest divisors of $n$ are $n,\frac{n}{p},\frac{n}{q}$ for prime $2\le p<q$ or $n,\frac{n}{p},\frac{n}{p^2}$ for prime $p\ge 2$. If the three largest divisors of $n$ are $n,\frac{n}{p},\frac{n}{q}$, then $\frac{\frac{n}{p}}{\frac{n}{q}}\in\mathbb{N}\Longleftrightarrow\frac{q}{p}\in\mathbb{Z}\Longleftrightarrow p\mid q$, contradiction. So the three largest divisors of $n$ are $n,\frac{n}{p},\frac{n}{p^2}$ for prime $p\ge 2$. In the similar spirit, we end up showing that $n=p_{k-1}$, which indeed works for any prime $p\ge 2$. Done. $\blacksquare$